What is the optimal number of items to produce for maximum profit in business?

Click For Summary
To maximize profit, a manufacturer needs to determine the optimal number of items to produce based on the cost function C(n) = n^2 - 6n + 35 and revenue R(n) = 50n. The profit function is derived as Profit = 50n - (n^2 - 6n + 35), simplifying to Profit = 56n - n^2 - 35. The derivative dP/dn = 56 - 2n is set to zero, leading to n = 28, which was initially thought to be the maximum. However, the correct approach involves recognizing that the cost function must account for the number of items produced, revealing that the optimal production quantity is actually 5 items. Understanding the cost per item is crucial for accurate profit maximization.
Shakattack12
Messages
24
Reaction score
2

Homework Statement


A manufacturer makes a batch of n items with the cost (in dollars) of each item being:
n2-6n+35.
The manufacturer sells the items for $50 each. How many items should be produced in each batch to maximise profit.

Homework Equations


Cost = C(n) = n2-6n+35
Revenue = R(n) = 50n

The Attempt at a Solution


Profit = 50n - (n2-6n+35)
= 56n - n2-35
dP/dn = 56 - 2n
let dP/dn = 0
0 = 56 - 2n
n = 28
After testing the nature I found it to be a maximum which is all good but the textbook says the answer is 5. Where did I go wrong?
 
Physics news on Phys.org
Shakattack12 said:

Homework Statement


A manufacturer makes a batch of n items with the cost (in dollars) of each item being:
n2-6n+35.
The manufacturer sells the items for $50 each. How many items should be produced in each batch to maximise profit.

Homework Equations


Cost = C(n) = n2-6n+35
Revenue = R(n) = 50n

The Attempt at a Solution


Profit = 50n - (n2-6n+35)
= 56n - n2-35
dP/dn = 56 - 2n
let dP/dn = 0
0 = 56 - 2n
n = 28
After testing the nature I found it to be a maximum which is all good but the textbook says the answer is 5. Where did I go wrong?

Your ##p = n^2 -6n + 35## is the cost per item, so the cost of a batch of ##n## items is ##C(n) = n p##.
 
Thank you so much! Can't believe I missed that.
 
Question: A clock's minute hand has length 4 and its hour hand has length 3. What is the distance between the tips at the moment when it is increasing most rapidly?(Putnam Exam Question) Answer: Making assumption that both the hands moves at constant angular velocities, the answer is ## \sqrt{7} .## But don't you think this assumption is somewhat doubtful and wrong?

Similar threads

Replies
1
Views
2K
  • · Replies 5 ·
Replies
5
Views
8K
Replies
2
Views
7K
Replies
1
Views
3K
  • · Replies 4 ·
Replies
4
Views
6K
Replies
6
Views
2K
  • · Replies 2 ·
Replies
2
Views
3K